Add recitation 2 problems for Phys 101.
[course.git] / latex / problems / Serway_and_Jewett_8 / problem03.43.tex
1 \begin{problem*}{3.43}
2 You are standing on the ground at the origin of a coordinate system.
3 An airplane flies over you with constant velocity parallel to the $x$
4 axis at a fixed height of $7.60\E{3}\U{m}$.  At time $t=0$, the
5 airplane is directly above you so that the vector leading from you to
6 it is $\vect{P}_0=7.60\E{3}\jhat\U{m}$.  At $t=30.0\U{s}$, the
7 position vector leading from you to the airplane is
8 $\vect{P}_{30}=(8.04\E{3}\ihat+7.60\E{3}\jhat)\U{m}$ as suggested in
9 Figure P3.43.  Determine the magnitude and orientation of the
10 airplane's position vector at $t=45.0\U{s}$.
11 \begin{center}
12 \begin{asy}
13 import Mechanics;
14
15 real u = 1cm;
16 real h = 1u;
17 real d = 8.04/7.6*h;
18 real dx = 0.2u;
19
20 draw((-dx,0)--(d+dx,0));
21 draw((-dx,h)--(d+dx,h));
22
23 Vector A = Vector();draw((0,0)--(0,h));
24 A.draw();
25 Vector B = Vector();((0,0)--(d,h));
26 B.draw();
27 \end{asy}
28 \end{center}
29 \end{problem*}
30
31 \begin{solution}
32 \end{solution}